86 + 16.95 - 3-0042
PLEASE HELP ME​

Answers

Answer 1

Answer:

86 + 16.95 - 3 - 0042

86 + 16.95 = 102.95

102.95 - 3 - 0042 = 57.95


Related Questions

Please help thanks this is due today!

Answers

D. [tex] ({ - 8})^{2} - 9 + ( - 7)[/tex] ✅

Step-by-step explanation:

[tex] {( - 32 \div4 })^{2} - 9 + ( - 7) \\ \\ = ({ - 8})^{2} - 9 +(- 7 )\\ \\=64-9-7\\\\ = 64 - 16 \\ \\ = 48[/tex]

Note:-

[tex]\sf\red{PEMDAS\: rule.}[/tex]

P = Parentheses

E = Exponents

M = Multiplication

D = Division

A = Addition

S = Subtraction

[tex]\sf\red{(+\:x\:-)\:=\:-}[/tex]

[tex]\sf\red{(-\:x\:-)\:=\:+}[/tex]

[tex]\bold{ \green{ \star{ \orange{Mystique35}}}}⋆[/tex]

URGENT PLEASE HELP WILL GIVE BRAINLEIST !!!!!!!!!!

Answers

Answer:

4

Step-by-step explanation:

1 change in x and 4 change in y from 1,0 to 0,4 or 4/1

Take any 2 points from the line and use the rule to find the slope

Slope=y2-y1/x2-x1
(0,4)(-1,0)

Slope= 0-4/-1-0 = -4/-1 = 4

Slope =4

I hope that answers your question and please brainliest my answer I would really appreciate it and best of wishes!!

Please help find the volume by using 3.14 as pi

Answers

Answer:

P

v = 794.0275 in³

Q

v = 1,307.81 in³

Step-by-step explanation:

volume of cone

v = πr²h

----------------------------

cylinder P

v = 3.14 * (4.25²) * 14

v = 794.0275 in³

-----------------------------

cylinder Q

v = 3.14 * (7²) * 8.5

v = 1,307.81 in³

please help i have no clue what i’m doing

Answers

Answer:

x = 24

x = 20

Step-by-step explanation:

x/3 - 2 = 6

Add 2 to both sides

x/3 = 8

multiply both sides by 3

x = 24

-----------------------------

x/5 +1 = 5

Subtract 1 from both sides

x/5 = 4

multiply both sides by 5

x = 20

Answer:

1. add two both sides and then multiply by 3 each side to get to the answer of 24.

Check: 24/3 = 8 - 2 = 6

2. x/5 + 1 = 5

subtract 1 from each side and then multiply by 4 on each side to get to the result of 20.

check: 20/5 is 4 + 1 = 5

Brainlist please :)

Anne is painting her house light blue. To make the color she wants, she must add 3 cans of white paint to every 2 cans of blue paint. How many cans of white paint will she need to mix with 6 cans of blue?

Answers

Answer:

9 cans of white paint

Step-by-step explanation:

To solve this you can set up a ratio.

The ratio right now is 3 : 2, you have 3 cans of white paint for every 2 cans of blue paint. This question is asking how many white paint cans would be needed for 6 cans of blue paint. You can see the relationship between the number of blue paint cans and the new number of blue paint cans, maybe it's multiplying by 4 or 2 for example, and once we find that out we can do the same exact thing to the white cans.

[tex]\frac{3}{2} = \frac{?}{6}[/tex]

We can see that to get from 2 to 6 you multiply by 3, so now we do that to the other side of the fraction as well, we multiply by 3. 3 multiplied by 3 is 9, so if we were to have 6 cans of blue paint we would need 9 cans of white paint to get that perfect shade of light blue. Anne would need 9 cans of white paint if she had 6 cans of blue paint to make her shade of blue.

Need help on number 3 please!!

Answers

Answer:

f(x) - g(x) = x - 3

Step-by-step explanation:

f(x) - g(x)    starting with the basic equation

2x - (x + 3)    implementing the values into equation

x - 3       subtracting x and 3 from 2x making 2x into 1x (or just x) and making a       negative 3

A calculator which is marked at $120 is sold for cash at a 30% discount. How much change would Susan receive if she pays for the calculator with a $100 bill?

Answers

Answer:

64 dollars

Step-by-step explanation: 120×30%= $36-$100= $64

She receives $16 when she pays for the calculator with a $100 bill

What is the percentage?

A percentage is a value per hundredth. Percentages can be converted into decimals and fractions by dividing the percentage value by a hundred.

Given, A calculator which is marked at $120 is sold for cash at a 30% discount.

So, It was sold for 70% of its mark value assuming that to x.

(70/100)×120 = x.

x = $84.

So, Susan will receive $(100 - 84) = $16.

learn more about percentages here :

https://brainly.com/question/24159063

#SPJ2

A Group of students is tracking a friend, John, who is riding a Ferris wheel. They know that John reaches the Maximum height of 11 m at 10 seconds and then reaches the minimum height of 1 m at 55 seconds.
What is the period of the function and what does it represent in this situation?

Answers

Answer:  Period = 90 seconds

The period represents how long it takes go from one max to the next max (ie peak to peak). It also represents going from min to min.

======================================================

Explanation:

It reaches the max at 10 seconds, and then it reaches the min at 55 seconds. This is a timespan of 55-10 = 45 seconds. This is exactly half the period. It doubles to 2*45 = 90 seconds. This is the time period between any two points in time when you're at the max height (from one max to the next adjacent max).

So you're at the max height at 10 seconds, then min at 55 seconds, then the max again at 100 seconds. Note how 100-10 = 90.

The period of any trig function is basically telling us when the function or cycle repeats itself. Once we reach the peak again, the ride repeats itself from before.

-----------

Notes:

90 seconds = 1 minute, 30 seconds = 1.5 minutesThe actual heights listed (11 m and 1 m for the max and min respectively) do not play a role in determining the period. So the heights can be anything we want, and the period will stay at 90 seconds.

Some triangles are drawn on squared paper. Which 3 triangles are congruent to one another?

Answers

Answer:

a, e, and d are all congruent

Step-by-step explanation:

they r all the same size and lengths

Those three triangles are; A,D,E

How? well see the attachment for this, have tried to make you understand.

Type the height of the ramp (h), in inches. Round your answer to the nearest whole inch.

Answers

Answer:

49 inches

Step-by-step explanation:

To obtain the height, h of the ramp ;; we use trigonometry ;

h = opposite and the value of the hypotenus :

Sin θ = opposite / hypotenus

Sin 22 = h / 132

h = 132 * sin 22

h = 132 * 0.3746065

h = 49.44

Height of ramp = 49 inches

PLS HELP!!!
The function c = 1.50(n – 2) + 5.50 represents the cost c in dollars of printing n invitations. Which of the following is not true?
A. Each invitation costs $5.50 to print.
B. For each additional invitation, it costs an extra $1.50 to print.
C. One can not print just one invitation.
D. The cost depends on the number of invitations printed.

Answers

Answer:

A

Step-by-step explanation:

Hope this helped!

Answer:

i am pretty sure its B

Step-by-step explanation:

Anyone help me please. A circle has a circumference
of 36. What is the radius of
the circle? Take pi to be 3.142. State the answer to one decimal
place.​

Answers

The circumference of a circle is pi x 2 x radius.

To find the radius, divide the circumference by pi, then divide that by 2:

36/3.142 = 11.4576703

Radius = 11.4576703/2 = 5.7288

Rounded to one decimal place the radius is 5.7 units.

Answer:

[tex]R=5.7[/tex]

Step-by-step explanation:

[tex]Radius=\frac{Circumference}{2\pi }[/tex]

[tex]R=\frac{36}{2(3.142)}[/tex]

[tex]R=\frac{36}{6.284}[/tex]

[tex]R=5.728835137[/tex]

[tex]R=5.7[/tex]

Hope this helped :)

I need help on this

Answers

The answer is FG = GI

Both of the lines formed by these points equal the same distance, which is 3 units.

Answer:

it is FH < HI

Step-by-step explanation:

I need help please and urgent!

Answers

Answer:

the answer is blurry for me take a picture again

Step-by-step explanation:

What value of g makes the equation true?

Answers

Answer:

3

Step-by-step explanation:

g = 7-4 = 3

=> x²+3x -28

__

PLEASE HELP 50 points

Answers

Answer:

Scalene triangle [any side and angle are not equal]isosceles triangle [two side and two angle are equal]equilateral triangle [all sides and angle are equal]right angled isosceles triangle [having one angle 90° and two other side and two angle are equal]scalene triangleequilateral triangleright angled trianglescalene triangleright angled isosceles triangleright angled isosceles trianglescalene triangleisosceles triangleequilateral triangleright angled triangleright angled isosceles triangleisosceles trianglescalene triangleequilateral triangleright angled isosceles triangleisosceles triangleequilateral trianglescalene triangleisosceles triangleright angled isosceles triangleright angled triangleright angled isosceles triangleisosceles triangle

helppppppp pllzzzzzzzzzzzz

Answers

The answer is D. Hope this helps

Answer:

D 1/25

Step-by-step explanation:

5 multiplied by the negative second power is .04, .04 times 25 equals 1.

65 70 75 80 85
Brand A
HIH
15
50
60
70
85
Brand B
0 5 10 15 20 25 30 35 40 45 50 55 60 65 70 75 80 85 90 95 100
Price ($)
Which statement is the most appropriate comparison of the spreads?
O A. The interquartile range (IQR) for brand A, $20, is less than the IQR
for brand B. $70.
O B. The median for brand A, $75, is greater than the median for brand
B, $60.
O C. The interquartile range (IQR) for brand A, $10, is less than the IQR
brand B. $20.
O D. The interquartile ranges (IQRs) for brands A and B are both $20.

Answers

Given:

Two box plots for Brand A and Brand B.

To find:

The correct statement for the given box plots.

Solution:

In a box plot, the left end of the box represents the first quartile [tex](Q_1)[/tex], right end of the box represents the third quartile [tex]Q_3[/tex] and the line inside the box represents the median.

The interquartile range of a data set is:

[tex]IQR=Q_3-Q_1[/tex]

From the box plot of Brand A, it is clear that

[tex]Q_1=70[/tex]

[tex]Median=75[/tex]

[tex]Q_3=80[/tex]

So, the interquartile range (IQR) for brand A is:

[tex]IQR=80-70[/tex]

[tex]IQR=10[/tex]

From the box plot of Brand B, it is clear that

[tex]Q_1=50[/tex]

[tex]Median=60[/tex]

[tex]Q_3=70[/tex]

So, the interquartile range (IQR) for brand B is:

[tex]IQR=70-50[/tex]

[tex]IQR=20[/tex]

The median for brand A, $75, is greater than the median for brand B, $60.

The interquartile range (IQR) for brand A, $10, is less than the IQR

brand B. $20.

Therefore, the correct options are B and C.

Why is 31
less than 3 x 10?

Answers

Answer:

Step-by-step explanation:

Because 3x10=30 and 31 is greater than 30 by 1 more.

It's not. 3 times 10 is 30.

30<31

Reflection: Across the line y = -1 ​

Answers

The A’ is your reflected shape. The red line is y=-1, or your line  right reflection.  To solve, I counted the distance from the red line on the original triangle, and then the same on the opposite side of the red line, and counted the same distance, plotting the points at the final spot.

You might need:

Calculator

Kenji runs a channel where he uploads and shares videos that he makes. He noticed an exponential

relationship between how long his videos have been posted and the number of times each video has been

viewed.

Kenji took the natural logarithm for the numbers of views only, and he noticed a linear relationship

between the how long his videos have been posted and the transformed numbers of views

Here's the least-squares regression equation for the transformed data, where "time" represents days since

posting, and "views" is the number of views

In(views) = 0.81(time) + 3.15

According to this model, what is the predicted number of views on a video that's been posted for 3 days?

You may round your answer to the nearest whole number.

views

Show Calculator

Answers

Using the logarithmic function, it is found that the predicted number of views of the video after 3 days is of 57.

The function that represented the relationship between the time in days and the number of views is given by:

[tex]\ln{v} = 0.81\ln{t} + 3.15[/tex]

We want the number of views after 3 days, thus, [tex]t = 3[/tex], and we have to solve for v.

[tex]\ln{v} = 0.81\ln{3} + 3.15[/tex]

[tex]\ln{v} = 0.89 + 3.15[/tex]

[tex]\ln{v} = 4.04[/tex]

[tex]e^{\ln{v}} = e^{4.04}[/tex]

[tex]v = 57[/tex]

57 views.

A similar problem is given at https://brainly.com/question/24286043

PLS HELP ME IM RLLY STUCK I WILL GIVE BRAINLIEST

Answers

Check the picture below.

PlzzPlease help me bunnies <3
No monkey business plz. Or I will report

Defined the explicit section for the sqeucrce in the table
Thank you

Answers

Answer:

D) [tex]a(n)=7+(n-1)*12[/tex]

Step-by-step explanation:

It is D since you can replace [tex]n[/tex] for any value on the [tex]x[/tex] side of the table and it will give you its corresponding [tex]y[/tex] value.

I think the answer is D

Help me please. NO LINKS

Answers

Answer:

A. 7986.45

[tex] \frac{4}{3} \pir3[/tex]

Please help!
The graphs of f(x)=−2x and g(x)=(12)x are shown.

What are the solutions to the equation −2x=(12)x ?

Select each correct answer.


-2

-1

2

4

Answers

Answer:

-1 and -2

Step-by-step explanation:

The function 2 represented by the equation f(x) = -(x - 2)² + 5 has the larger maximum.

What is function?

Function, in mathematics, an expression, rule, or law that defines a relationship between one variable (the independent variable) and another variable.

The functions are given as:

Function 1

f(x) = -(x - 4)² + 1

Function 2:

f(x) = -(x - 2)² + 5

A quadratic function is represented as a (x - h)² + k, when a is negative then, the vertex of the function is a maximum. In both functions, the value of a is -1. This means that both functions are at a maximum

The values of k in both functions are k = 1 and k = 5. By comparison, 5 is greater than 1. Therefore, function 2 has larger maximum.

Read more about quadratic functions at:

brainly.com/question/4140287

#SPJ3

complete question;

The following graph describes function 1, and the equation below it describes function 2. Determine which function has a greater maximum value, and provide the ordered pair. Function 1 graph of function f of x equals negative x squared plus 8 multiplied by x minus 15 Function 2 f(x) = −x2 + 2x − 15 Function 1 has the larger maximum at (4, 1). Function 1 has the larger maximum at (1, 4). Function 2 has the larger maximum at (−14, 1).

oWhich value completes this table?

Answers

Answer:

When x = -1, f(x) = 2

Step-by-step explanation:

f(x) = (1/2)^x

When x = -4

f(x) = (1/2)^x

f(x) = (1/2)^-4

= 1 ÷ (1/2)^⁴

= 1 ÷ 1^⁴/2^⁴

= 1 ÷ 1/16

= 1 × 16/1

= 16

When x = -1

f(x) = (1/2)^x

= (1/2)^-1

= 1 ÷ 1/2^1

= 1 ÷ 1/2

= 1 × 2/1

= 1 × 2

= 2

When x = -1, f(x) = 2

Suppose that the monthly salary can be modeled by the function p(x) = 20,000 + 50x. In the formula, 20,000 represents the base salary, and the person gets an additional $50 for every sale. The variable x represents the number of sales. How many items does the salesperson need to sell to earn $30,000?

Answers

Answer:

$50 is the earned for every sales

$30000 is earned for 30000/50=600

He has to made 600 sales to earn 30000

Hi, My answer is correct but I want to know how did I found 72 as the correct answer. Thanks already :) and please don't put any link

Answers

Answer:

Lol its just 48 divided by 2/3 and u get 72

Step-by-step explanation:

If you can give Brainiest answer it would be much help :)

When f(n)=12, what is the value of n?

Answers

The correct answer is simply 12.

Pls help me with this

Answers

Answer:

44ft

Step-by-step explanation:

12ftx6ft=72ft

72ftx2ft= 144ft

144ft-100ft=44ft

Hope this helps! :)

L x W x H
Length = 12
Width = 6
Height = 2

The answer is 144
Other Questions
Consider a figure in a coordinate plane. For each of the transformations below, first, transform the figure as stated. Then reverse the order of the sentences and transform the original figure a second time. Did the sequences result in the same image or a different image? Drag and drop each transformation in the cell with the appropriate heading What can an author do to promote a reader's suspension of disbelief?A. Describe the magical or supernatural elements of the story andhow they affect the charactersB. Help the reader feel a sense of belonging or attachment to a groupdescribed in a storyC. Use a pattern of writing in which words or phrases are similar informatting and structureD. Focus on a story's realistic, ordinary details instead of itssupernatural elements2 If two heterozygous individuals are crossed, whatpercent of their offspring are also expected to beheterozygous?Complete the punnet square plssssss Can anyone please help me with this ? Thanks 2.5m = 75 What is most likely Mr. White's final wish?A. To stop his wife from feeling so sadB. To go back in time and never make the first wishC. To return the 200 pounds he receivedD. To reverse his second wish Question 5Find the volume. ANSWER PLEASEEEE!!!!!!!!!!! I need help with number so can any one help me please and thanks Question:What is the molar concentration of 1.29 mol of KCL dissolved in 350 mL of solution? i need help with this question three-quarter the product of two numbers x and y, what is the algebriac expression? As an estimation we are told 3 is 4.Convert 33 to euros. A woman received 5 per cent interest on a loan of $300 for 1 year. How much interest did she receive? $5 $15 $30 $150 none of these Who was the first American to orbit the earth?John ShepardJohn GlennBill ArmstrongO Neil Armstrong How can magnetic levitation be improved? Whats the best cereal ? what is it called when a seismic wave bends as it passed from liquid to solid HELP ME ASAP!!! Tristan is stacking building blocks to make a tower. He takes a break when the tower is 5 2/5 feet tall, which is 1/3 of the height of the tower he wants to build. How tall is the tower when finished? Check all that applyA. 1/3 x ? = 5 2/5B. 1/3 5 2/5 = ?C. 5 2/5 x ? = 1/3D. 5 2/5 x 1/3 = ?E. 5 2/5 x 3/1 = ?F. 5 2/5 1/3= ? Why do you suppose none of the prisoners harbor any thoughts ofrevenge after their liberation? A dolphin is 9 feet below the surface of the ocean. If its position can be recorded as 9 feet, what would the position of 0 represent?